4
$\begingroup$

Let $d$ be a large positive integer. Let $x$ be uniformly distributed on the unit-sphere in $\mathbb R^d$ and let $a$ and $b$ be perpendicular vectors in $\mathbb R^d$, i.e such that $a^\top b=0$. Let $f:\mathbb R \to \mathbb R$ be a continuous function which is twice-continuously differentiable at $0$ (you may assume more regularity if that helps).

Question. How large can the absolute difference $$ \Delta(a,b)=|\mathbb E[f(x^\top a)f(x^\top b)] - \mathbb E[f(x^\top a)]\mathbb E[f(x^\top b)]| $$ be as a function of the dimension $d$ ?

Observation 1. In the case where $x \sim N(0,I_d)$, or more generally, with any diagonal covariance matrix, we have $\Delta(a,b) = 0$. Indeed, in this case, the joint distribution of $(x^\top a,x^\top b)$ is $N(0, C)$, where $C$ is the $2 \times 2$ psd matrix with entries given by

  • $c_{11}=\mathbb E[(x^\top a)^2] = \mbox{trace}(aa^\top)=\|a\|^2$,
  • $c_{12}=c_{21}=\mathbb E[(x^\top a)(x^\top b)] = \mbox{trace}(ab^\top) = a^\top b = 0$,
  • $c_{22}=\mathbb E[(x^\top b)^2] = \mbox{trace}(bb^\top)=\|b\|^2$.

Thus, $x^\top a$ and $x^\top b$ are jointly gaussian random variables with zero correlation, and so are independent. Thus $\Delta(a,b) = 0$.

Observation 2. Since $d$ is large both $x^\top a$ and $x^\top b$ are close to centered normal random variables with variance $\|a\|^2/d$ and $\|b\|^2/d$ respectively, so maybe an adaptation of the previous argument might be of use ?

$\endgroup$

2 Answers 2

3
$\begingroup$

$\newcommand{\de}{\delta}\newcommand{\ep}{\varepsilon}$The covariance of two random variables (r.v.'s) does not change if one of them is shifted by a constant. So, without loss of generality $f(0)=0$. Let $n:=d$.

To compute the asymptotics, we need to assume that \begin{equation*} f(x)=Ax+Bx^2+Cx^3+Dx^4+e_1x^5+O(x^6) \tag{1} \end{equation*} for some real $A,B,C,D,e_1$ and all real $x\in[-1,1]$.

The joint distribution of $x^\top a$ and $x^\top b$ is the same as that of $Y_1$ and $Y_2$, where \begin{equation*} Y_j:=X_j/|X|, \end{equation*} $X_1,\dots,X_n$ are iid standard normal r.v.'s and $|X|:=\sqrt{X_1^2+\dots+X_n^2}$. So, we want to find the asymptotics of \begin{equation*} \de:=Ef(Y_1)f(Y_2)-(Ef(Y_1))^2 \tag{2} \end{equation*} (as $n\to\infty$). Note that \begin{equation*} EY_1^2=\dots=EY_n^2=\frac1n, \tag{3} \end{equation*} since the $Y_i$'s are exchangeable and $Y_1^2+\dots+Y_n^2=1$. Also, using e.g. a Chernoff exponential concentration inequality for $|X|$, we see that $|X|$ is highly concentrated near $\sqrt n$, say in the sense that, for each real $\ep>0$,
$P(|\,|X|-\sqrt n|>\ep\sqrt n)$ goes to $0$ faster than any negative power of $n$. It follows that for each real $p>0$ \begin{equation*} E|Y_1|^p\sim E|X_1|^p n^{-p/2}. \tag{4} \end{equation*}

So, by (1) and the symmetry of (the distribution of) $Y_1$,
\begin{equation*} Ef(Y_1)=\frac Bn+\frac{(3+o(1))D}{n^2}. \tag{5} \end{equation*}

Next, by (1), the symmetry of the $Y_i$'s, and (4) \begin{equation*} Ef(Y_1)f(Y_1)\\ =B^2\,EY_1^2Y_2^2+2BD\,EY_1^2Y_2^4+o(n^{-3}). \tag{6} \end{equation*} Similarly to (4), \begin{equation*} EY_1^2Y_2^4\sim EX_1^2X_2^4 n^{-3}=\frac3{n^3}. \tag{7} \end{equation*}

The main difficulty here is to estimate $EY_1^2Y_2^2$. We have \begin{equation} EY_1^2Y_2^2=Eh(G), \tag{8} \end{equation} where $G^2$ has the $\chi^2$ distribution with $n-2$ degrees of freedom and \begin{align*} h(x)&:=E\frac{X_1^2X_2^2}{(X_1^2+X_2^2+x^2)^2} \\ &=\frac1{2\pi}\int_0^{2\pi}dt\int_0^\infty r\,dr\,e^{-r^2/2}\frac{r^4\cos^2t\sin^2t}{(r^2+x^2)^2} \\ &=\frac1{16}\int_0^\infty du\,\frac{u^2e^{-u/2}}{(u+x^2)^2} \\ &=\frac{x^2}{16}\int_0^\infty dt\,\frac{t^2e^{-x^2 t/2}}{(1+t)^2}. \end{align*} Writing $1/(1+t)^2=1-2t+O(t^2)$ for $t\in(0,1)$, we now get \begin{equation} h(x)=\frac1{16}\,(x^{-4}-(12+o(1))x^{-6}) \tag{9} \end{equation} as $x\to\infty$. Also, \begin{equation} EG^{-4}=\frac1{n^2}+\frac{10+o(1)}{n^3},\quad EG^{-6}=\frac{1+o(1)}{n^3}. \end{equation} Hence, in view of (9), \begin{equation} EY_1^2Y_2^2=Eh(G)=\frac1{n^2}-\frac{2+o(1)}{n^3}. \end{equation}

Finally, recalling (2), (5), and (7), we get \begin{equation} \de=-\frac{2B^2+o(1)}{n^3}. \end{equation} So, as should be expected, we have a small negative correlation between $x^\top a$ and $x^\top b$ (on the order of $1/n^2$, in view of (3)).

$\endgroup$
1
  • $\begingroup$ Thanks very much. $\endgroup$
    – dohmatob
    Apr 14, 2021 at 16:24
1
$\begingroup$

It turns out that one can get a stronger result than demanded in the question: compute $\Delta(a,b)$ for any $a,b \in S_{d-1}$, perpendicular or not. Indeed,

Claim. If $f(\rho)=a_0 + a_1 \rho + a_2 \rho^2 + a_3 \rho^3 + \mathcal O(\rho^4)$, then for every $u,v \in S_{d-1}$, and $x$ be uniform on the sphere, then we have the following approximation \begin{eqnarray} \begin{split} \mathbb E_x[f(x^\top u)f(x^\top v)]-\mathbb E_x[f(x^\top u)]\mathbb E_x[f(x^\top v)] &= -\frac{a_2^2}{d^2}\\ &+(\dfrac{a_1^2}{d}+\frac{6a_1a_3}{d^2})u^\top v\\ & + \dfrac{2a_1^2}{d^2}(u^\top v)^2\\ &+\mathcal O(\dfrac{1}{d^3}). \end{split} \end{eqnarray} In paricular, if $u^\top v = 0$, then $$ \mathbb E_x[f(x^\top u)f(x^\top v)]-\mathbb E_x[f(x^\top u)]\mathbb E_x[f(x^\top v)] = -\frac{a_2^2}{d^2}+\mathcal O(\frac{1}{d^3}), $$ as proved by Iosif Pinelis (in the accepted answer).

For the proof, we will need the following result

Lemma. Let $u$ and $v$ be fixed and $x$ be uniformly random on the unit-sphere. Let $p$ and $q$ be nonnegative integers and define $c_{p,q}(u,v):=\mathbb E_x[(x^\top u)^p(x^\top v)^q]$. If $p$ and $q$ have different parities, then $c_{p,q}(u,v)=0$. Otherwise, we have the formula \begin{eqnarray} c_{p,q}(u,v) = \dfrac{p!q!\Gamma(\frac{d}{2})}{2^{p+q}\Gamma(\frac{d+p+q}{2})} \sum_t \dfrac{2^t}{t!(\frac{p-t}{2})!(\frac{q-t}{2})!}(u^\top v)^t, \end{eqnarray} where the sum is over all $t$ between $0$ and $p \land q$ inclusive, that have the same parity as $p$ and $q$. The formula is simplified in the table below for special values of $p$ and $q$. The above lemma is proved (ME link) here https://math.stackexchange.com/a/4004804/168758.

Proof of the claim. WLOG, let $a_0=0$. Thanks to the lemma, one may compute $$ (\mathbb E_x[f(x^\top u)])^2 = (\frac{a_2}{d}+\mathcal O(\frac{1}{d^2}))^2 = \frac{a_2^2}{d^2} + \mathcal O(\frac{1}{d^3}), $$ and similarly \begin{eqnarray*} \begin{split} \mathbb E_x[f(x^\top u)f(x^\top v)] &= \sum_{p=0}^3\sum_{q=0}^3 a_p a_q\mathbb E_x[(x^\top u)^p(x^\top v)^q]+\mathcal O(\dfrac{1}{d^3})\\ &= \dfrac{a_1^2}{d}u^\top v + \dfrac{2a_2^2}{d(d+2)}(u^\top v)^2 + \dfrac{6 a_1a_3}{d(d+2)}u^\top v+\mathcal O(\dfrac{1}{d^3}) \\ &=(\dfrac{a_1^2}{d}+\frac{6a_1a_3}{d^2})u^\top v + \dfrac{2a_2^2}{d^2}(u^\top v)^2 +\mathcal O(\dfrac{1}{d^3}), \end{split} \end{eqnarray*} and the claim follows after subtracting the previous display. $\quad\quad\quad\quad\Box$

$\endgroup$

Your Answer

By clicking “Post Your Answer”, you agree to our terms of service and acknowledge you have read our privacy policy.

Not the answer you're looking for? Browse other questions tagged or ask your own question.